Difference between revisions of "2021 AMC 10B Problems/Problem 9"

m (Problem)
(Redirected page to 2021 AMC 12B Problems/Problem 5)
(Tag: New redirect)
 
Line 1: Line 1:
==Problem==
+
#redirect [[2021 AMC 12B Problems/Problem 5]]
The point <math>P(a,b)</math> in the <math>xy</math>-plane is first rotated counterclockwise by <math>90\deg</math> around the point <math>(1,5)</math> and then reflected about the line <math>y = -x</math>. The image of <math>P</math> after these two transformations is at <math>(-6,3)</math>. What is <math>b - a ?</math>
 
 
 
<math>\textbf{(A)} ~1 \qquad\textbf{(B)} ~3 \qquad\textbf{(C)} ~5 \qquad\textbf{(D)} ~7 \qquad\textbf{(E)} ~9</math>
 
==Solution==
 
D
 

Latest revision as of 18:40, 11 February 2021